Nursing 216-Test 1

Pataasin ang iyong marka sa homework at exams ngayon gamit ang Quizwiz!

Which of the following can the nurse tell a patient about antipyretic drugs during fever? a) "They are not recommended." b) "Antipyretics are ineffective in most cases." c) "They usually have little effect." d) "Antipyretics help to protect the body."

"Antipyretics help to protect the body." Antipyretic drugs are given to alleviate the discomfort of fever and protect vulnerable organs, such as the brain from extreme elevations in body temperature. They are usually effective

A student states, "It seems like helper T cells do a lot more than just 'help' the cellular immunity process". Which of the following responses listed below best conveys an aspect of the role of CD4+ helper T cells in immunity? a) "Helper T cells are key to the hematopoiesis that produces all the components of the immune system." b) "Helper T cells play a major role in stimulating and regulating the whole process." c) "Without helper T cells, the wrong antibodies would end up being produced." d) "Without helper T cells, no antigens would be presented."

"Helper T cells play a major role in stimulating and regulating the whole process." Helper T cells are central to the regulation, proliferation and stimulation of the immune system. They do not play a central role in antigen presentation or early hematopoiesis, however. Their absence would not result in incorrect antibody production, but rather insufficient or absent immune response.

A student states, "It seems like helper T cells do a lot more than just 'help' the cellular immunity process". Which of the following responses listed below best conveys an aspect of the role of CD4+ helper T cells in immunity? a) "Helper T cells are key to the hematopoiesis that produces all the components of the immune system." b) "Helper T cells play a major role in stimulating and regulating the whole process." c) "Without helper T cells, no antigens would be presented." d) "Without helper T cells, the wrong antibodies would end up being produced."

"Helper T cells play a major role in stimulating and regulating the whole process." Explanation: Helper T cells are central to the regulation, proliferation and stimulation of the immune system. They do not play a central role in antigen presentation or early hematopoiesis, however. Their absence would not result in incorrect antibody production, but rather insufficient or absent immune response.

A nurse educator is describing the way that cells involved in the inflammatory response find their way to the site of injury. Which description best reflects this physiologic mechanism? a) "Phagocytosis is the dynamic and energy-directed process where cells migrate, directed by chemoattractants." b) "The process of margination ensures that cells will follow the cytokine gradient." c) "The process of chemotaxis is the process where cells wander through the tissue guided by secreted chemoattractants." d) "Tissues have an abundance of inflammatory cells that are constantly migrating and just waiting for tissue injury."

"The process of chemotaxis is the process where cells wander through the tissue guided by secreted chemoattractants." Chemotaxis is the dynamic and energy-directed process of directed cell migration. Once leukocytes exit the capillary, they wander through the tissue guided by a gradient of secreted chemoattractant, bacterial and cellular debris, and protein fragments generated from activation of the complement system. -Phagocytosis is the engulfment of foreign or infective material. -Margination is the first step of the cellular stage and is a separate process from chemotaxis, which is the third step in the cellular response. -Tissues do not store inflammatory cells.

Apoptosis

"cell suicide" or programmed death

Prodrome

- First period of fever where mild headache and fatigue occurs. General malaise (discomfort) and pains.

Endothelial Cells

- Make up the single-cell-thick lining of blood vessels. - Helps to separate the intravascular and extravascular space. - Contain vasodilators and constrictors. - Semi-permeable

Platelets (Thrombocytes)

- Membrane bound discs circulating in the blood that play an active role in normal hemostasis. - Inflammatory mediators

Leukocytes (WBC)

- Neutrophils: 60-70%. Engulfs bacteria. - Eosinophils: 2-3%. Highly toxic to parasites that cannot be phagocytize. - Basophils or mast cells: <1%. Help in allergic reactions. Releases histamine. - Monocyte: Help in healing tissues but can also phagocytize. - Lymphocytes: B and T. Activates macrophages. Produce antibodies.

Stages of Fever

- Prodrome - Chill (Uncomfortable sensation of being cold and shaking) - Flush: Cutaneous vasodilation occurs and skin becomes warm and reddened. - Defervescence: Initiation of sweating.

Cardinal Signs of Inflammation (5)

- Redness -Swelling - Heat - Loss of Function - Pain

Fever Patterns

- Sustained or continuous - Recurrent or Relapsing: Cycles that can last as long as several days. - Intermittent: Temp returns to normal at least once every 24 hours. (sepsis) - Remittent: Temp does not return to normal but varies a few degrees in either direction.

Two stages of acute inflammation

- Vascular: Increased blood flow and vascular permeability. - Cellular: Immune response and other cells coming into the action.

Proliferative phase of cutaneous wound healing

-2-3 days from injury until 3 weeks -Primary purpose: building of new tissue to fill wound space (granulation tissue)--fibroblasts -Epithelialization

Inflammatory phase of cutaneous wound healing

-Begins at time of injury -Prepares wound environment of healing -Includes hemostasis and cellular phase of inflammation

Keloid

-Benign, tumor-like masses caused by excess production of scar tissue -Develop genetically, more common in African American and dark skin -Usually just a cosmetic effect, but can become large and limit mobility

Infection

-Prolongs inflammatory phase -Impairs function of granulation tissue -Inhibits proliferation of fibroblasts and deposition of collagen

What happens during the emigration of fibroblasts and deposition of extracellular matrix phase?

-Scar formation builds on the granulation tissue framework of new vessels and loose ECM -2 phases: emigration of fibroblasts and deposition of ECM

A nurse is assessing a client for the classic signs of acute inflammation. The nurse would assess the client for: a) Parasthesis, redness, and coolness b) Pain, pulselessness, and edema c) Rubor, swelling, and pain d) Cyanosis, heat, and swelling

Rubor, swelling, and pain The classic signs of inflammation are rubor (redness), tumor (swelling), calor (heat), and dolor (pain). -The remaining options are more characteristic of symptomatology resulting from circulatory dysfunction.

A client has a watery fluid leaking from a site of inflammation. The nurse would document this type of exudate as: a) Fibrinous b) Serous c) Suppurative d) Hemorrhagic

Serous Serous exudate is a watery fluid low in protein content that results from plasma entering the inflammatory site. -Hemorrhagic exudate is red or blood tinged related to damage to blood vessels. -Suppurative exudate is composed of degraded white blood cells and tissue debris, leaving the fluid pus-like. -Fibrinous exudate is thick and sticky meshwork fluid.

A client is being evaluated for atopic dermatitis possibly caused by a latex-related allergic reaction. The nurse will review which of the following labs to determine if an allergy is present? a) Basophils b) Serum IgG c) Serum IgE d) Neutrophils

Serum IgE IgE binds to mast cells and basophils and is involved in parasitic infections, allergic and hypersensitivity reactions. Serum IgE causes the symptoms of allergic reactions and is elevated in type 1 hypersensitivity disorders.

Example of Metaplasia

Smokers cells change from ciliated to stratified squamous which causes a chronic cough

The nurse knows the cells primarily programmed to remove the invading organisms and remember the antigen to respond rapidly during the next exposure are: a) CD4 and CD8 cells. b) Natural Killer (NK) cells and macrophages. c) White blood cells and platelets. d) T-and B-lymphocytes.

T-and B-lymphocytes. T- and B- lymphocytes are unique in that they are the only cells in the body capable of recognizing specific antigens present on the surfaces of microbial agents and other pathogens.

______ ______ is a rare autosomal recessive disorder that primarily affects families of Eastern Jewish origin.

Tay-Sachs Disease

A patient is experiencing anorexia, myalgia, arthralgia, headache, and fatigue. The nurse should assess for which of the following? a) Respirations b) Hypothermia c) Urinary output d) Temperature

Temperature Common clinical manifestations of fever include anorexia, myalgia, arthralgia, headaches, and fatigue; thus, the nurse should assess the patient's temperature.

Manifestations of NF-2 include changes in the ______, affecting ______.

retinas, vision

A hospital client who is being treated for acute kidney injury has developed an oral temperature of 39.1°C (102.4°F) despite the absence of other signs and symptoms of infection. When attempting to determine the etiology of the client's fever, the nurse should: a) auscultate the client's apical heart rate for at least 1 minute. b) measure the client's lying, sitting, and standing blood pressure. c) auscultate the client's lungs and facilitate lung function testing. d) review the client's recent medication history.

review the client's recent medication history. Drugs are a frequent cause of fevers that are unrelated to infection. For this reason, a review of the client's medication history would likely be prioritized over vital signs, provided the client is not in immediate distress.

Reasons for Apoptosis

signaling factor of cell damage receptors, mitochondrial damage, protein p53 activated by DNA damage

innate defense

skin and mucus membranes also cells and chemicals in body fluids

Neurofibromas usually form on or just beneath the ______, in the ______, and ______ ______ system.

skin, brain, peripheral nervous

In Chronic Hex-A deficiency, the affected individual may experience ______ speech.

slurred

Reasons for Necrosis : Liquefaction

some cells die, catylitic enzymes dont get destroyed and cause an absess

Manifestations of NF-1 include abnormal development of the ______, ______, or ______.

spine, skull, tibia

One symptom of Tay-Sachs Disease is that affected individual may develop a ______ ______.

staring gaze

Congenital defects are abnormalities of the body's ______, ______, or ______.

structure, function, metabolism

immunology

study of adaptive immunity (ability of an organism to recognize and defend itself against infectious agents) and how the immune system responds to specific infectious agents and toxins.

Bone Marrow Biopsy

study of the marrow's overall cellularity detection of focal lesions.

A gene mutation can involve the ______, ______, or ______ of one or two base pairs of DNA.

substitution, insertion, deletion

Treatment techniques for Neurofibromatosis include ______ to ______ tumors.

surgery, remove

Manifestations of Marfan Syndrome may include crowded ______ due to high ______.

teeth, palate

pathophysiology

the physiology of altered health, greek word pathos means disease

immunity

the protection from disease and infectious diseases

lymphoid organs

tonsil, adenoids, lymph nodes

Gas gangrene

toxic bubbles, clostridium infection produces toxins and H2S bubbles

true or false? Necrotic tissue cannot be reversed

true

Type 2-NF is characterized by slow growing ______ on the ___th ______ nerve.

tumors, 8, cranial

NF-1 is characterized by the presence of ______, ______ abnormalities, and changes in ______ appearance.

tumors, bone, skin

Manifestations of NF-2 include developing ______ in the ______ system.

tumors, nervous

How does apoptosis work?

turn their own enzymes on themselves, digest their own cell, proteins, and DNA, then are destroyed by white blood cells

Necrosis

unregulated cell death caused by injury to the cell

Stressed cells may fill up with ..

unused foods, abnormal protein, pigmenet, calcium salts

The prognosis of NF-2 ______ greatly among individuals.

varies

pathogen that is always intracellular

viruses

Biological agents

viruses and bacteria- diverse array of mechanisms

Type 1-NF is also known as "______."

von Recklinghausen Disease

In Chronic Hex-A deficiency, the affected individual may experience difficulty ______ due to ______, ______ ______, and ______ ______

walking, weakness, muscle cramps, decreased coordination

Scenario: Mr.X had a stroke and blood flow is cut off to part of the brain- Why give him oxygen?

we want to hyperoxygenate so that cells can have as much oxygen as possible

There are ______ ______ options for Hex-A deficiency.

zero treatment

A patient with a rising temperature is pale and has begun to shiver. The nurse reports that the patient is in which of the following phases of fever development? a) Defervescence b) Prodrome c) Chill d) Flush

Chill During the second phase or chill phase of fever development, the patient's skin is pale; there is an onset of shivering, a rising temperature, and the sensation of being chilled. Therefore, the nurse should report that the patient is in the second or chill phase of fever development.

Elderly are more vulnerable to _________ wounds

Chronic (pressure sores, etc.)

A client with end-stage renal disease received a kidney transplant with a kidney donated by a family member. The client has been carefully monitored for signs of rejection. The physician informs the client that there has been a gradual rise in the serum creatinine over the last 5 months. What type of rejection does this depict? a) Acute antibody-mediated rejection b) Chronic rejection c) Hyperacute rejection d) Acute rejection

Chronic rejection Chronic rejection involves immune-mediated inflammatory injury to a graft that occurs over a prolonged period. It is most often due to the inability to maintain adequate immunosuppression necessary to control residual circulating antigraft T lymphocytes or antibodies. Hyperacute rejection occurs almost immediately after vascular reperfusion to graft tissue occurs. Acute rejection and acute antibody=mediated rejection occur days to weeks after implantation.

______ Tay-Sachs Disease is a ______ but ______ form of Hex-A Deficiency.

Chronic, rare, milder

Which client manifestation indicates signs of drug fever? a) Temperature reaches 40°C (104°F) every afternoon, pulse 76 beats/minute, pruritis b) Temperature reaches 38°C (100.4°F) for 3 days, pulse 106 beats/minute, purulent drainage c) Temperature reaches 39°C (102.2°F) following aerobic activity, pulse 125 beats/minute, sweating d) Temperature reaches 41°C (105.8°F) after a sauna, pulse 76 beats/minute, skin hot and dry

Temperature reaches 40°C (104°F) every afternoon, pulse 76 beats/minute, pruritis Clients with drug fever often show signs of hypersensitivity such as joint pain, itching, rashes, muscle pains, and gastrointestinal distress. The fever has a diurnal pattern with the highest point in the afternoon or early evening. The clients in the other options are exhibiting signs of infection, heat exhaustion, and heatstroke.

The loss of heat from the body through the circulation of air currents is known as which of the following? a) Radiation b) Convection c) Evaporation d) Conduction

Convection Convection refers to heat transfer through the circulation of air currents, while - radiation is the transfer of heat through air or a vacuum. -Conduction is the direct transfer of heat from one molecule to another, and -evaporation involves the use of body heat to convert water on the skin to water vapor.

One week after a client received a transplant the nurse notes that the skin shows a maculopapular rash on the hands and feet. The client reports itching and nausea. What is the likely cause of these manifestations? a) Immunodeficiency disorder b) Anaphylactic reaction c) Opportunistic infection d) Graft versus host disease

Graft versus host disease Graft versus host disease is a serious condition in which the recipient is "attacked" by the graft cells. Three conditions are necessary for this to occur. The client must be immunologically compromised, the recipient's cells must secrete antigens that are not part of the donor cells, and the donor graft must contain cells that are immunologically capable of attacking the host cells. The condition is more likely to develop when there is not a close tissue match. Early signs include rash and itching, and progress to nausea, diarrhea, and liver disease. If gastrointestinal bleeding occurs, it indicates mucosal ulceration. The more severe the reaction, the lower the rate of survival.

A nurse's hand-off reports states that the patient has pyrexia. The nurse plans care for the patient with which of the following events? a) Fever b) Rash c) Wound d) Incontinence

Fever When a patient is documented to have pyrexia, the nurse should plan care for a patient with a fever.

A client is being treated for a pressure ulcer and the care team has observed that the wound is healing. Which activity will take place during the proliferative phase? a) White blood cell migrate the site of the client's wound b) Scar tissue is synthesized and becomes visible at the wound site c) Blood clots form at the wound site to prevent bleeding d) Fibroblasts secrete the cells necessary for wound healing

Fibroblasts secrete the cells necessary for wound healing The primary processes during the proliferative phase focus on the building of new tissue to fill the wound space. The key cell during this phase is the fibroblast, a connective tissue cell that synthesizes and secretes the collagen, proteoglycans, and glycoproteins needed for wound healing. White cell migration and clot formation take place during the inflammatory phase. Scar formation occurs during remodelling.

A client has an increase in core body temperature. What assessment findings does the nurse expect? a) Flushed skin b) Decreased skin temperature c) Blue nail beds d) Decreased urination

Flushed skin The client with an increase in their core temperature will be accompanied by flushed, warm skin as the body tries to lower the temperature. -The other assessments do not correlate with increased core temperature.

Bone Marrow Aspirate Analysis

Focuses on cellular morphology and determination of a differential cell count.

The nurse assessing a client admitted with a diagnosis of "fever of unknown origin" notes that the temperature is elevated, but the heart rate is within normal range. What further assessment will the client require? a) V/Q scan for pulmonary emboli b) Thyroid scan for nodules c) Test for Legionnaire disease d) Sedimentation rate for RA

Test for Legionnaire disease Typically, the heart rate rises with an elevation of temperature. If the heart rate is more rapid than anticipated, it could be from hyperthyroidism or pulmonary emboli. A heart rate that is slower than expected could result from Legionnaire disease or drug fever. Further testing is needed to confirm the cause of the fever and determine proper treatment.

Can a primary intention wound switch to secondary intention?

Yes, if it gets infected

Which client most likely faces the highest risk of neuroleptic malignant syndrome?

Young adult client who is taking antipsychotic medications for the treatment of schizophrenia. Antipsychotics are often implicated in cases of neuroleptic malignant syndrome. Surgery, medications for Alzheimer disease and fever are not risk factors.

A gene mutation is a biochemical event that results in...

a change in the DNA sequence of a gene

An example of a stressed cell filling up with pigment is

a tattoo, black lung, jondus

Some members of the population are so sensitive to certain antigens that they react within minutes by developing itching, hives, and skin erythema, followed shortly thereafter by bronchospasm and respiratory distress. What is this near immediate reaction commonly known as? a) Anaphylactic reaction b) Hyposensitive reaction c) Arthus reaction d) Antigen reaction

a) Anaphylactic reaction Anaphylaxis is a systemic life-threatening hypersensitivity reaction characterized by widespread edema, vascular shock secondary to vasodilation, and difficulty breathing. It is not called an antigen reaction, neither is it called an Arthus reaction.

A 48 year-old man who has been HIV positive for 6 years has just learned that he has been diagnosed with Kaposi sarcoma (KS). Which of the following facts most accurately conveys an aspect of his diagnosis? a) He is likely to have lesions on his skin, mouth or GI tract. b) Heterosexual contact most likely underlies his HIV and subsequent KS. c) An opportunistic Epstein-Barr virus underlies the man's KS. d) Intense pain was probably his first manifestation of KS

a) He is likely to have lesions on his skin, mouth or GI tract. The lesions of KS can be found on the skin and in the oral cavity, gastrointestinal tract, and the lungs. More than 50% of people with skin lesions also have gastrointestinal lesions. It is linked with a herpes virus and can often be painless, especially in early stages. Men who have sex with men are at a higher risk of developing KS

The nurse is aware that the only class of immunoglobulins to cross the placenta is: a) IgG b) IgM c) IgA d) IgD

a) IgG IgG is the only class of immunoglobulins to cross the placenta. Levels of maternal IgG decrease significantly during the first 3 to 6 months of life, while infant synthesis of immunoglobulins increases.

Three days ago, a mother delivered her full-term infant who had been identified as having an in utero infection. The infant is receiving antibiotic and phototherapy, and the mother is breastfeeding. Which of the following types of immunoglobulins could most reasonably be expected to predominate in the infant's immune system? a) IgG, IgA, IgM b) IgA, IgM, IgD c) IgM, IgD, Igm d) IgE, IgG, IgD

a) IgG, IgA, IgM Infants are born with IgG from transfer across the placenta, while IgA is found in colostrum. IgM is indicative of an in utero infection.

A woman experiences a viral infection while pregnant. Which of the following types of immunity does an infant have at birth against this infection? a) Passive b) Active c) Adaptive d) Tolerance

a) Passive Active immunity is acquired through exposure to antigens and having the B and T lymphocytes develop a response. Passive immunity is when the antibodies against an antigen are transferred directly to the host, such as when a fetus gains from the mother's immune system. Tolerance is the ability of the immune system to react to foreign substances but not the body cells. Adaptive immunity is when the immune system responds to antigens and is moderated by B and T lymphocytes.

The nurse is teaching a group of parents about foods that commonly cause allergic reactions in children. The most important information to provide would be: Select all that apply. a) Peanuts b) Shellfish c) Eggs d) Strawberries e) Milk

a) Peanuts b) Shellfish c) Eggs e) Milk The foods most commonly causing these reactions are milk, eggs, peanuts, tree nuts, fish, and shellfish (i.e., crustaceans and mollusks).

The first physical line of defense in innate immunity is: a) Skin and mucous membranes b) Plasma proteins c) Neutrophils d) Specialized lymphocytes

a) Skin and mucous membranes The first physical line of defense are the skin and mucous membranes, which prevent pathogens from entering. Plasma proteins, neutrophils, and specialized lymphocytes are examples of cellular level protection.

Stem cells in the bone marrow produce T lymphocytes or T cells and release them into the vascular system. The T cells then migrate where to mature? a) Thymus b) Spleen c) Pancreas d) Liver

a) Thymus The T lymphocytes (T cells) are generated from stem cells in the bone marrow and complete their maturation in the thymus and function in the peripheral tissues to produce cell-mediated immunity, as well as aid antibody production.

Tay-Sachs Disease is caused by an ______ of the enzyme ______.

absence, Hex-A

What happens when calcium is released into the cell?

acts as a "second messenger system" , turns on intracellular enzymes, can open more calcium gates in cell membrane

Type 2-NF usually develops tumors in ______ patients.

adult

Cellular Adaptation

allows the stressed tissue to survive or maintain function

A genetic disorder is a disease caused by...

an abnormality in the DNA

Manifestations of Marfan Syndrome may include dilation of the ______.

aorta

Babies with PKU are usually ______.

asymptomatic

Example of Hypertrophy

athletes muscles get bigger

An ______ ______ ______ is a pattern of inheritance in which the affected individual has one copy of a mutant gene and one normal gene on a pair of autosomal chromosomes.

autosomal dominant disorder

An ______ ______ ______ is a pattern of inheritance in which each parent contributes the defective gene.

autosomal recessive disorder

adaptive defense

b and t cells

Hypoxia

deprives the cell of oxygen and interrupts ATP generation

2 major functions of immune system

destroy pathogens and detect and kill abnormal cells

PKU can be ______ to the ______ system, causing ______ ______.

destructive, nervous, mental retardation

Marfan Syndrome may result in ocular abnormalities such as lens ______, retinal ______, and ______.

displacement, detachment, blindness

Chemical Agents

drugs, lead, mercury, caron monoxide, insecticides, tobacco smoke- liver damage , injure the cell membrane, block enzymatic pathways, coagulate cell proteins, disrupt the ionic and osmotic balance

The onset of an autosomal recessive disorder is usually ______ in life.

early

monocyte/macrophage

engulf and kill invading organisms, and function as an antigen presenting cells of the adaptive immune response

Nutritional imbalances

excess or deficiency of a nutrient- anemia

True or false? Cells usually maintain high intracellular calcium levels

false ; low

Which temperature readings indicate to the nurse that the clients have fever? Select all that apply. a) 35-year-old with pulmonary artery reading of 37.9ºC (100.2ºF) b) 19-year-old with oral reading of 38.4ºC (101.1ºF) c) 3-day-old with rectal reading of 38ºC (100.4ºF) d) 2-year-old with axillary reading of 35.8ºC (96.4ºF) e) 77-year-old with tympanic reading of 36.3ºC (97.3ºF)

• 35-year-old with pulmonary artery reading of 37.9ºC (100.2ºF) • 19-year-old with oral reading of 38.4ºC (101.1ºF) • 3-day-old with rectal reading of 38ºC (100.4ºF) A core body temperature reading can be obtained from the esophagus, pulmonary artery catheter, a urinary catheter probe, or the rectum. For rapidly fluctuating temperatures, the pulmonary artery is most accurate. An oral temperature is generally lower than core by about 0.5ºC (1ºF). Axillary temperatures are approximately 0.5ºC (1ºF) lower than oral. A fever is temperature elevation above the body's normal set point which is usually between 36ºC (97ºF) and 37.5ºC (99.5ºF).

2 main lines of defense

innate (internal external) defense and adaptive defense

In Chronic Hex-A deficiency, the affected individual may have changes in ______, ______, or ______.

intellect, hearing, vision

One symptom of Tay-Sachs Disease is the affected individual may stop ______ with ______.

interacting, others

Dry gangrene

lack of arterial blood supply but can carry fluid out of the tissue, causes tissue to coagulate

Wet gangrene

lack of veneous flow so a bunch of fluid accumulates; organ is cold and swollen; foul odor

Manifestations of NF-1 include ______-colored spots on the skin.

latte

adaptive immunity

less rapid but more effective, focuses on recognition of each foreign agent and comes up with a response to defend. (Can be different, specific )

Calcium cascade

letting in more calcium to the cell

Electricity

lightning or high-voltage wires- extensive tissue injury or disruption of neural and cardiac impulses

Example of Atrophy

menopause- decrease in hormones which causes a decrease in reproductive organ size.

In Chronic Hex-A deficiency, the affected individual may develop ______ illness.

mental

The manifestations of Tay-Sachs Disease are progressive ______ ______ and ______ ______.

mental retardation, physical regression

Epithelialization

migration, proliferation and differentiation of epithelial cells at wounds' edges to form new surface layer **requires a moist, vascular wound site

Free radicals

molecules with an unpaired electron in the outer electron electron shell ; they are extremely unstable and reactive, can damage normal cell component and produce more.

NF-1 is ______ ______ than NF-2.

more common

Marfan Syndrome is a ______ disease that affects the ______, ______, and ______ systems.

multi-systemic, skeletal, ocular, circulatory

PKU is caused by the ______ of a gene on chromosome #___.

mutation, 12

Abnormalities that may cause genetic disorders range from a small ______ in a single ______ to the ______ or ______ of an entire ______.

mutation, gene, addition, subtraction, chromosome

Tay-Sachs Disease causes progressive destruction of ______ cells located in the ______ and ______ ______.

nerve, brain, spinal cord

Neurofibromatosis is caused by a mutation of a gene that codes for a protein called ______.

neurofibromin

Is there a cure for Marfan Syndrome? If so, what is it?

no

Reasons for Necrosis : Infarction

no blood supply

One symptom of Tay-Sachs Disease is normal levels of ______ will ______ the individual to an abnormal degree.

noise, startle

Scenario: 2 boys suffer from hypoxia : one with a normal body temp and one who is very cold - Which one will have a lower intracellular pH?

normal body temp because metabolism is the same with an increase in lactic acid

Most cases of NF-1 live ______ and ______ lives.

normal, productive

Antioxidants

normally remove free radicals from the body

extracellular

occurring outside of cells

intracellular

occurring w/i cells

Maturation and remodeling stage of wound healing

occurs when collagen synthesis= degradation

Accumulated damage in aging cells

older cells have more DNA damage, more free radicals, and cells lose their ability to repair their telomeres.

Manifestations of NF-1 include the presence of an ______ ______ tumor.

optic nerve

Free radical formation causes

oxidation

pathogen that is always extracellular

parasitic worms

5 types of pathogen largest to smalles

parasitic worms, fungi, protozoa, bacteria, viruses

The mutation responsible for PKU causes toxic levels of the amino acid ______ in the ______, poisoning ______ cells in the ______.

phenylalanine, blood, nerve, brain

The mutation that causes PKU occurs on the gene that codes for a ______ called ___ (phenylalanine hydroxylase), an ______ in the ______.

protein, PAH, enzyme, liver

A common form of treatment for PKU is a ______ ______.

protein-free diet.

A client has been diagnosed with osteomyelitis and admitted to the hospital. The client's fever persists throughout most of the day but returns to normal at least twice a day. Which pattern of fever is this client displaying?

Intermittent Intermittent fever patterns are very changeable, but they do return to normal at least once every 24 hours. -A remitting fever pattern temperature does not return to normal and varies a few degrees in either direction. - In a sustained fever pattern, the temperature remains above normal with minimal variations. -A relapsing fever is one in which there is one or more episodes of fever, each as long as several days, with 1 or more days of normal temperature between episodes.

Thalamus

Involved in the sensation and regulation of body temp.

Evaporation

Involves the use of body heat to convert water on the skin to water vapor.

Radiation

Involves transfer of heat through the air or a vacuum.

What will happen to a cell that does not make adaptive changes as a result of stress?

It will die

Which one of the following is often found in a patient with AIDS? a) Carcinoma of the lung b) Kaposi's sarcoma c) High peripheral blood CD4+ lymphocyte counts d) Overactive T-cell function

Kaposi's sarcoma Kaposi's sarcoma is an opportunistic infection with a virus that causes a tumor on the skin. It is commonly found in individuals who have a depressed immune system such as in the case of AIDS or immunosuppression therapy. An underactive T-cell function and low peripheral blood CD4+ lymphocyte counts are seen in patients with HIV/AIDS, not the opposite. A carcinoma of the lung is not a defining characteristic of HIV/AIDS, as an individual may have this carcinoma without having HIV/AIDS.

A 36-year-old male who is positive for HIV antibodies notices purplish spots on his upper body. Which of the following terms is used to identify these areas as an opportunistic infection? a) Kaposi's sarcoma b) Toxoplasmosis c) Multifocal leukoencephalopathy d) Liver spots

Kaposi's sarcoma (violet colored) Purplish spots identified in a HIV-positive person on the trunk, neck, or head are uniquely Kaposi's sarcoma, an opportunistic cancer that is 2000 times more likely to occur in immunocompromised persons. Toxoplasmosis and multifocal leukoencephalopathy are opportunistic infections but are not skin lesions. Liver spots occur with aging and sun exposure and are not solely related to those with HIV.

Gangrene

Large area of necrotic tissue

The nurse evaluating the bloodwork results of a client with an infected leg ulcer. The white blood cell count is 18,000 cells/uL. The nures inteprets thsi as: a) Leukocytosis b) Lymphadenitis c) Nuetropenia d) Lymphacytosis

Leukocytosis A white blood cell count of 18,000 cells/uL is indicative of an elevated white blood cell count or leukocytosis (normal range is 4000-10,000 cells/uL). This would be an expected finding in a client with an infected leg ulcer. -An increase in lymphocityes is lymphacytosis and neutropenia is a decrease in neutrophil's. Lymphadenitis is an inflammation in the lymph nodes.

A 53-year-old female hospital patient has received a kidney transplant following renal failure secondary to hypertension. As part of the teaching while she was on the organ wait list, she was made aware that she would need to take anti-rejection drugs for the rest of her life. Which aspect of the immune system underlies this necessity? a) MHC molecules will never develop in the cells of the donor organ and effector cells will be continually stimulated. b) Anti-rejection drugs will stimulate the production of familiar MHC molecules. c) Donor organ antibodies will be identified as foreign and stimulate an immune response. d) The lack of identifiable major histocompatibility complex (MHC) molecules will stimulate the innate immune response.

MHC molecules will never develop in the cells of the donor organ and effector cells will be continually stimulated. Explanation: The lack of familiar MHC molecules will stimulate an immune response by effector cells in the absence of anti-rejection drugs. An innate immune response is not central to the response, but rather the adaptive immune system. Lack of known MHC molecules, not foreign antibodies, accounts for the immune response, and familiar MHC molecules will not be produced by the donor kidney cells.

______ ______ is an autosomal dominant disorder which results from dysfunction of the gene that normally codes for the connective tissue protein ______.

Marfan Syndrome, fibrillin

Scenario: A woman breaks her leg, 3 wks. later you find .. a nodule of skin tissue has formed a fluid-filled cyst near her incision

Metaplasia

A client was skating on the lake when the ice broke and he was submerged for 15 minutes. When rescuers were able to retrieve him from the water, his core temperature was 31° C (87.8°F). Into which classification does this temperature fit? a) Lethally hypothermic b) Mildly hypothermic c) Moderately hypothermic d) Severely hypothermic

Moderately hypothermic Core body temperatures in the range of 32°C to 35°C (90°F to 95°F) are considered mildly hypothermic; 28°C to 32°C -(82.4°F to 90°F), moderately hypothermic; and -less than 28°C (82.4°F), severely hypothermic. -Lethally hypothermic is not a classification, but if hypothermia goes untreated, it is possible to freeze to death.

phagocytic cells

Monocytes/macrophages, granulocytes, dendritic cells.

Myalgia

Muscle pain

______ is an autosomal dominant genetic disorder characterized by the growth of noncancerous tumors called "______."

Neurofibromatosis, neurofibromas

A patient has a fever that was induced by damage to the hypothalamus due to intercranial bleeding. The nurse plans care for the patient with which of the following types of fever? a) Exdogenous b) Neurogenic c) Systemic d) Intrinsic

Neurogenic Neurogenic fever has its origin in the central nervous system and is usually caused by damage to the hypothalamus from trauma, intercranial bleeding, or increased intercranial pressure. The nurse should plan care for a patient with a neurogenic fever.

A patient presented to the emergency department of the hospital with a swollen, reddened, painful leg wound and has been diagnosed with methicillin-resistant Staphylococcus aureus (MRSA) cellulitis. The patient's physician has ordered a complete blood count and white cell differential. Which of the following blood components would the physician most likely anticipate to be elevated? a) Platelets b) Neutrophils c) Basophils d) Eosinophils

Neutrophils Increased neutrophils are associated with inflammation in general and bacterial infections in particular. -Platelets play a role in inflammation but their levels would not rise to the same extent as would neutrophils. -Eosinophils are not strongly associated with bacterial infection and -basophils would not increase to the same degree as neutrophils.

Typical symptoms seen in the latent period of HIV infection include which of the following? a) Skin lesions b) No signs or symptoms c) Night sweats d) Gastrointestinal problems

No signs or symptoms The primary phase is followed by a latent period during which the person has no signs or symptoms of illness

What is the most common cause of drug fever?

Hypersensitivity reactions.

Scenario: A woman breaks her leg, 3 wks. later you find .. the circumferance of the right calf has increased by 2 cm

Hypertrophy

Consanguineous mating ______ the chance of becoming a ______ of an autosomal ______ disorder.

increases, carrier, recessive

There are three types of Tay-Sachs Disease: 1)______, 2)______, and 3)______ (Adult-Onset)

infantile, juvenile, chronic

Cellular phase of inflammation

-phagocytic white blood cells -arrive within minutes and leave by 3-4 days -Macrophages come 24-48 and stay much longer

Wound healing in diabetes

-poor wound healing, collagen formation, tensile strength, and small blood vessel disease

Granulation tissue

-red, moist connective tissue that fills injured area while necrotic debris is removed -has capillaries, fibroblasts, and inflammatory cells

Remodeling phase of cutaneous wound healing (2 processes)

1. Synthesis of collagen and fibroblasts 2. lysis by collagenase enzymes

Proud flesh

-when excessive granulation tissue forms and extends above edges of wound preventing epithelialization -requires surgery or cauterization

Stresses damage cells by..

1) free radical formation 2) increased intracellular calcium 3) ATP depletion

In Infantile Tay-Sachs Disease, by age ___ the child may be completely ______.

1, blind

In Infantile Tay-Sachs Disease, by age ___ the child may develop an inability to ______ and ______.

1, breathe, swallow

In Infantile Tay-Sachs Disease, by age ___ the child's ______ will be quite ______.

1, head, large

In Infantile Tay-Sachs Disease, by age ___ the child may experience ______ as the disease progresses.

1, paralysis

In Infantile Tay-Sachs Disease, by age ___ the child will be very ______ with floppy ______.

1, weak, muscles

3 phases of connective tissue healing

1. Hemostasis, angiogenesis, and ingrowth of granulation tissue 2. Emigration of fibroblasts and deposition of extracellular matrix 3. Maturation and remodeling of fibrous tissue

3 phases of cutaneous wound healing

1. Inflammatory 2. Proliferative phase 3. Remodeling phase

Factors Affecting Wound Healing (3)

1. Nutritional status 2. Blood flow and oxygen delivery 3. Impaired inflammatory and immune responses (ex: diabetes)

In Juvenile Tay-Sachs Disease, death usually occurs by age ___.

15

Tay-Sachs Disease is caused by a defective gene on chromosome #___.

15

Type 1-NF is caused by a gene mutation on chromosome #___.

17

In Juvenile Hex-A deficiency, symptoms appear between the ages of ___ and ___, and the disease will progress more ______.

2, 5, slowly

In Infantile Tay-Sachs Disease, by age ___ the child may experience ______.

2, seizures

Onset of Marfan Syndrome is usually by age...

20

Type 2-NF is caused by a gene mutation on chromosome #___.

22

In an autosomal recessive disorder, two parents who are both carriers have a ___% chance of having a child who ______ the dissorder.

25, manifests

Approximately 1 in every ___ Jews in the United States is a ______ of the Tay-Sachs Disease gene.

27, carrier

Symptoms of Tay-Sachs Disease appear by ___ to ___ months of age when no new skills are being learned.

3 to 6

In Infantile Tay-Sachs Disease, death usually occurs between ___ and ___ years of age.

4, 5

A client has experienced an acute inflammatory response with an elevation of white blood cells. The nurse is reviewing the client's most recent lab results to determine if the counts have returned to a normal range. Select the result that suggests the client is now within normal range. a) 14,000 to 20,000 cells/μL b) 4000 to 10,000 cells/μL c) 1000 to 3000 cells/μL d) 1500 to 20,000 cells/μL

4000 to 10,000 cells/μL A normal value of white blood cells would be 4000 to 10,000 cells/μL. In acute inflammatory conditions, the white blood cell count commonly increases from 15,000 to 20,000 cells/μL. The other results are abnormal.

In Chronic Hex-A deficiency, symptoms may begin around age ___, or may not occur until age ___ to ___.

5, 20, 30

A parent affected with an autosomal dominant disorder has a ___% chance of transmitting the mutant gene.

50

A nurse is providing care for several clients on a neurological unit of a hospital. With which of the following clients would the nurse be justified in predicting a problem with thermoregulation? a) A 45 year old female with a T8 fracture secondary to a diving accident. b) A 22 year old male with damage to his cerebellum secondary to a motorcycle accident. c) A 68 year old male with end stage neurosyphilis. d) A 66 year old male with damage to his thalamus secondary to a cerebral vascular accident.

A 66 year old male with damage to his thalamus secondary to a cerebral vascular accident. The thalamus is involved in the sensation and regulation of body temperature. Syphilis, a T8 fracture and damage to the cerebellum would be unlikely to manifest by difficulties with thermoregulation.

Which symptom indicates the next stage of a fever after a prodrome? a) Predrome b) A chill c) A flush d) Defervescence

A chill The physiologic behaviors that occur during the development of fever can be divided into four successive stages: prodrome, chill, flush, and defervescence. The stages are successive.

A client has been diagnosed with a FUO fever of unknown cause; the nurse recognizes this as: a) A fever of new onset b) A fever that only occurs in adults c) A prolonged fever that does not have an identified source d) A fever that has responded well to several medications

A prolonged fever that does not have an identified source A prolonged fever for which the cause is difficult to ascertain is often referred to as fever of unknown origin (FUO). FUO is defined as a temperature elevation of 38.3°C (101°F) or higher that is present for 3 weeks or longer. An FUO can occur in children or adults.

When counseling a male patient with suspected HIV, the nurse informs him that if the enzyme-linked immunosorbent assay (ELISA) come back positive, then: a) He will be sent to an infectious disease physician for a tissue biopsy to confirm infection. b) If the second test, the Western blot, returns negative, he has not developed a case of full-blown AIDS. c) A second test known as the Western blot assay will be ordered to confirm positive HIV status. d) No further testing is required since this confirms HIV infection.

A second test known as the Western blot assay will be ordered to confirm positive HIV status. If ELISA positive, then sent for Western blot assay. If Western blot is positive, diagnosis of HIV confirmed. If Western blot is negative, then the person is NOT infected with HIV.

A client presents with an oral temperature of 38.7°C and painful, swollen cervical lymph nodes. Laboratory results indicate neutrophilia with a shift to the left. Which diagnosis is most likely? a) A severe bacterial infection b) A localized fungal infection c) A mild parasitic infection d) A mild viral infection

A severe bacterial infection Fever and painful, palpable lymph nodes are nonspecific inflammatory conditions; -leukocytosis is also common but is a particular hallmark of bacterial infection. -Neutrophilia also indicates a bacterial infection, -whereas increased levels of other leukocytes would indicate other etiologies. -The shift to the left---the presence of many immature neutrophils---indicates that the infection is severe, because the demand for neutrophils exceeds the supply of mature cells.

A 24-year-old woman presents with fever and painful, swollen cervical lymph nodes. Her blood work indicates neutrophilia with a shift to the left. She most likely has: a) A severe fungal infection b) A severe bacterial infection c) A mild viral infection d) A mild parasitic infection

A severe bacterial infection Fever and painful, palpable lymph nodes are nonspecific inflammatory conditions; leukocytosis is also common but is a particular hallmark of bacterial infection. Neutrophilia also indicates a bacterial infection, whereas increased levels of other leukocytes would indicate other etiologies. The shift to the left--the presence of many immature neutrophils--indicates that the infection is severe, because the demand for neutrophils exceeds the supply of mature cells.

A client is brought to the physician's office with a raised red macular rash on the trunk and arms accompanied by a fever. A diagnosis of measles is made. Which type of immunity does this disease process provide? a) Passive artificial immunity b) Passive natural immunity c) Active natural immunity d) Active artificial immunity

Active natural immunity Active immunity is acquired when the host mounts an immune response to an antigen either through the process of vaccination (artificial means) or from environmental exposure (natural means). It is called active immunity because it requires the host's own immune system to develop an immunological response.

Acute vs Chronic

Acute - Short duration Chronic - Long duration

Which of the following clients is at the greatest risk for developing an intracellular pathogen infection? a) A hyperthyroid client who has received treatment with radioactive iodine b) An AIDS client with a decreased CD4+ TH1 count c) A teenager who attends a crowded high school d) A breast cancer client who has a WBC count of 8000

An AIDS client with a decreased CD4+ TH1 count Thus, the CD4+ TH1 cell controls and coordinates host defenses against certain intracellular pathogens, a function that helps to explain why a decreased CD4+ TH1 count in persons with acquired immunodeficiency syndrome (AIDS) places them at high risk for intracellular pathogen infections. If the teenager has a healthy immune system, high school should not place him or her at high risk for infection. A WBC of 8000 is normal. Radioactive iodine attacks the thyroid gland and does not usually interfere with immunity

An 88 year old resident of a long term care home has been suffering from a three day onset of increasing shortness of breath and decreased oxygen saturation. At the hospital, an anterior-posterior chest X-ray and sputum culture and sensitivity has confirmed a diagnosis of bacterial pneumonia, yet the client's tympanic temperature has not exceeded 37.3°C (99.2°F). The health care team would recognize that which of the following phenomena likely underlies this situation? a) An older adult is often insensitive to exogenous pyrogens. b) An older adult is sometimes incapable of vasodilation. c) An older adult's hypothalamus has diminished thermoregulatory ability. d) Infections manifest by cognitive changes in older adults.

An older adult's hypothalamus has diminished thermoregulatory ability. The hypothalamus in older adults is often less capable of thermoregulation than in younger clients. -There are sometimes alterations in the release of endogenous pyrogens and deficits in vasoconstriction. -While infections do often manifest with cognitive changes in older adults, this does not explain why fever is precluded.

Scenario: A woman breaks her leg, 3 wks. later you find .. The left leg is smaller than the right

Atrophy

Select the statement that best describes autoimmune disease. a) Autoimmune diseases results from an overuse of antibiotics that causes damage to body tissues by the immune system. b) Autoimmune diseases represent a disruption in self-tolerance that results in damage to body tissues by the immune system. c) Autoimmune diseases represent overuse of the immune system that causes damage to the body tissues. d) Autoimmune diseases represent an increase in self-tolerance that results in damage to body tissues by the immune system.

Autoimmune diseases represent a disruption in self-tolerance that results in damage to body tissues by the immune system. Autoimmunity results from a failure of tolerance. Autoimmune disorders may be triggered by environmental stimuli, such as infections, in a genetically predisposed individual. Overuse of antibiotics, however, does not lead to autoimmune diseases.

Select the type of lymphocyte that matures in the bone marrow. a) Macrophages b) Cytotoxic T lymphocytes c) B lymphocytes d) T lymphocytes

B lymphocytes B lymphocytes mature in the bone marrow and are essential for humoral or antibody-mediated immunity. The T lymphocytes mature in the thymus. Macrophages are part of the monocytic phagocyte system and are in almost all tissues and are the mature form of monocytes. Cytotoxic T cells are involved in cell-mediated immunity.

Normal Values of white Blood Cells

Between 4000 and 10,000.

Type 2-NF is also known as "______."

Bilateral Acoustic NF

Anticholinergic

Blocks neurotransmitter acetylcholine in the central and peripheral nervous system. Blocks nerve impulses of parasympathetic nervous system.

Dysplasia

Change in size, shape, and organization

Metaplasia

Change in the type of cell from one type to another

Abscess

Collection of pus that is restricted to a specific area.

A lactation nurse visits a new mother after delivery of her first child and encourages the mother to breastfeed her infant, even for a short time. Which statement made by the nurse correctly explains the importance of breastfeeding? a) Colostrum will provide the infant with innate immunity to diseases to which the mother is immune. b) Colostrum will provide the infant with passive immunity to all childhood illnesses for several months. c) Colostrum will provide the infant with passive immunity for several months to diseases to which the mother has immunity. d) Colostrum will provide the infant with active immunity to many childhood illnesses for several years.

Colostrum will provide the infant with passive immunity for several months to diseases to which the mother has immunity. After birth, the neonate receives IgG antibodies from the mother in breast milk or colostrum. Therefore, infants are provided with some degree of protection from infection for approximately 3 to 6 months, giving their own immune systems time to mature. IgA is primarily a secretory Ig that is found in saliva, tears, colostrum (i.e., first milk of a nursing mother), and bronchial, gastrointestinal, prostatic, and vaginal secretions. Its primary function is in local immunity on mucosal surfaces. IgA prevents the attachment of viruses and bacteria to epithelial cells

What is the leading cause of infant death?

Congenital defects

A patient has a splinter. The nurse expects a granulomatous inflammatory response. What does this involve? a) Macrophages attacking the splinter in order to dissolve it b) Connective tissue encapsulating and isolating the splinter c) Normal inflammatory mechanisms acting on the splinter d) Splinter being digested and dissolved

Connective tissue encapsulating and isolating the splinter Granulomatous inflammatory response to a splinter would involve the connective tissue encapsulating and isolating it for removal. Each of the other options are part of the normal inflammatory response, and would not apply in this situation.

In providing education to a sexually active 22-year-old female, which of the following would be most beneficial to discuss to decrease her risk for contracting HIV? a) Use of petroleum (oil based) lubricants to help kill the virus b) Use of natural condoms with sexual activity c) Consistent use of latex condoms with sexual activity d) Not engaging in sexual activity until marriage

Consistent use of latex condoms with sexual activity The use of latex condoms is the best defense against contacting HIV. Natural or lambskin condoms and oil-based lubricants are not recommended. As an educator you should not place your values on the patient and advising an already sexually active female to practice abstinence is not the best choice.

Hemostasis

Constriction of injured blood vessels and initiation of clotting **After a period of this, some vessels actually dilate and increase permeability to allow blood to leak into injured area

Purulent Exudate

Contains pus, composed of white blood cells, proteins and tissue debris.

A nurse's hand-off reports states that the patient has pyrexia. The nurse plans care for the patient with which of the following events? a) Fever b) Incontinence c) Wound d) Rash

Fever When a patient is documented to have pyrexia, the nurse should plan care for a patient with a fever.

One way to diagnose Chronic Hex-A deficiency is to perform ______ ______ to check for ______.

DNA tests, carriers

Wound healing in aging skin

Decreased dermal thickness, collagen content, and elasticity causes impaired wound healing

An example of a stressed cell filling up with unused foods is

Fatty liver and Taysacs disease

An older adult's dysfunctional temperature regulator function places the client at greatest risk for: a) Delayed initiation of appropriate treatment b) Acute renal failure c) Acute dementia d) Misdiagnosis of pathology

Delayed initiation of appropriate treatment The elderly often have a lower baseline temperature (36.4°C [97.6°F]) than younger persons, and although their temperature increases during an infection, it may fail to reach a level that is equated with significant fever. The absence of fever may delay diagnosis and initiation of antimicrobial treatment. Absence of fever does not result in misdiagnosis. While the remaining options may occur, they are not the greatest risks for injury.

Conduction

Direct transfer of heat from one molecule to another. (blood transfers heat from core to skin)

Supportive treatment for Marfan Syndrome includes ______, annual ______ exams, heart ______ and blood ______ repair.

EKGs, eye, valve, vessel

The nurse is assessing a client for acute inflammation of a wound. For which of these symptoms of infection does the nurse assess? a) Tissue necrosis b) Hypothermia c) Edema d) Pallor

Edema Cardinal signs of inflammation include rubor (redness), tumor (swelling or edema), calor (heat), dolor (pain) and functio laesa (loss of function). -Tissue necrosis occurs with chronic inflammation

Select the statement that best describes the effectiveness of vaccination in the elderly population.

Experimental evidence suggests that vaccination is less successful in inducing immunization in older persons than in younger adults. Elderly persons tend to be more susceptible to infections, have more evidence of autoimmune and immune complex disorders than younger persons, and have a higher incidence of cancer. Experimental evidence suggests that vaccination is less successful in inducing immunization in older persons than in younger adults. However, the effect of altered immune function on the health of elderly persons is clouded by the fact that age-related changes or disease may affect the immune response.

Angiogenesis

Growth of new capillaries

One way to diagnose PKU is a ______ test for ______ gene.

Guthrie, defective

A nurse is providing care for a 44-year-old male client who is admitted with a diagnosis of fever of unknown origin (FUO). Which characteristic of the client's history is most likely to have a bearing on his current diagnosis? a) Cachexic and Asian b) HIV positive and homeless c) Receiving intravenous normal saline with 20 mEq KCl d) Malnourished, hypomagnesemic, and hypocalcemic

HIV positive and homeless FUO is associated with HIV. The other aspects of the client's circumstances are not noted to correlate with FUO.

Which of the following serves as recognition markers to aid in self-tolerance? a) Macrophages of the immune system b) HLA antigens encoded by MHC genes c) Inactivated B cells d) Inactivated T cells

HLA antigens encoded by MHC genes The immune system must be able to differentiate foreign antigens from self-antigens in a process called self-tolerance. It is the HLAs encoded by MHC genes that serve as recognition markers of self and nonself for the immune system. To elicit an immune response, an antigen must first be processed by an antigen-presenting cell (APC), such as a macrophage, which then presents the antigenic determinants along with an MHC II molecule to a CD4+ helper T cell

The nurse is caring for a client whose temperature is increasing. The nurse is aware that the client will also experience an increase in: a) Blood pressure b) Respiratory rate c) Heart rate d) White blood cell count

Heart rate Critical to the analysis of a fever pattern is the relation of heart rate to the level of temperature elevation. Normally, a 1°-C rise in temperature produces a 15-bpm (beats/minute) increase in heart rate (1°F, 10 bpm). The remaining options are not as closely related as is heart rate.

Convection

Heat transfer through the circulation of air currents. Wind chill is an example.

Antipyretic Drugs

Help to alleviate the discomfort of fever and protect vulnerable organs. Usually effective.

Another name for Tay-Sachs Disease is ______ ______.

Hex-A deficiency

A client with environmental allergies is experiencing respiratory inflammation. Which mediator causes vasodilation during the vascular stage of the inflammatory response? a) Histamine b) Memory T cells c) Leukotrienes d) Adhesion molecules

Histamine Histamine is a key mediator in the inflammatory system and one that induces vasodilation during the vascular stage. Adhesion molecules, memory T cells, and leukotrienes do not participate in the process of vasodilation during the vascular stage.

When explaining what is occurring when their child has an acute bronchial asthma attack, the nurse will emphasize that which mediator is primarily responsible for the bronchial constriction? a) Histamine. b) Tree pollen. c) Mold dust. d) T-lymphocyte proliferation.

Histamine. Asthma response begins within 5-30 min. of exposure to an allergen. It is mediated by mast cell degranulation and the release of preformed and/or enzymaticaly activated mediators. These mediators include histamine, serotonin, acetylcholine ...Histamine is the most recognized mediator of type I hypersensitivity reactions and ultimately results in bronchial constriction.

Scenario: A woman breaks her leg, 3 wks. later you find .. new calluses on the right foot

Hyperplasia

What is the most common cause of drug fever? a) Serotonin syndrome b) Impaired peripheral heat dissipation by atropine c) Hypersensitivity reaction to medication d) Increased heat production from PTU

Hypersensitivity reaction to medication The most common cause of drug fever is a hypersensitivity reaction. Drug fever can also be caused by the antithyroid medication propylthiouracil (PTU), atropine and anticholinergic medications, antipsychotic agents, tricyclic antidepressants, cocaine, and amphetamines. -The agitation, hyperthermia, and hyperactivity of serotonin syndrome occur with overdose of serotonin reuptake inhibitors.

A patient with common variable immunodeficiency (CVID) is admitted to the hospital for treatment. Which of the following types of treatment will the nurse be administering to the patient? a) Bone marrow transplant b) IV immunoglobulin c) IV corticosteroids d) IV antibiotics

IV immunoglobulin Treatment methods for CVID are similar to those used for other primary humoral immunodeficiencies with IV Ig being the mainstay of therapy.

ATP depletion

If mitochondria is not getting enough oxygen = hypoxia

In a newborn, which antibody crosses the placenta to provide protection from infection? a) IgE b) IgG c) IgA d) IgM

IgG IgG antibodies cross the placenta to provide protection from infection. The other antibodies do not normally cross the placenta.

Eosinophilia

Increase in eosinophilia from parasitic and allergic infections.

Hyperplasia

Increase in number of cells

Neutrophilia

Increase in selective neutrophils from BACTERIAL infections.

Cold

Increase viscosity of blood and vasoconstriction

A client tells the nurse she is at risk to prematurely deliver her baby and is concerned that the baby does not have a developed immune system. The best response would be: a) All babies have deficient systems. b) Infants born prematurely may be deficient immunity. c) Antibiotics are available; do not worry. d) The immune system and protection are completed early in pregnancy

Infants born prematurely may be deficient immunity. The largest amount of IgG crosses the placenta during the last weeks of pregnancy and is stored in fetal tissues; therefore, infants born prematurely may be deficient.

Osteomyelitis

Infection of the bone. Rare but serious. Typically caused by a form of staph.

In which of the following patients with a transecting spinal cord injury should the nurse anticipate an impaired ability for temperature regulation? a) Injury at T2 b) Injury at T9 c) Injury at L1 d) Injury at L4

Injury at T2 Spinal cord injuries that transect the cord at T6 or above can seriously impair temperature regulation because the thermoregulatory centers in the hypothalamus can no longer control skin blood flow and sweating. T9, L4, and L1 injuries are below the level of T6 and do not impair the hypothalamus' ability to control skin blood flow and sweating.

A client asks why his temperature is always below 98.6°F. The nurse responds: a) A person's highest point of core temperature is usually first thing in the morning. b) The best way to bring your body temperature up to normal is to live in a warmer climate. c) Some people maintain a core body temperature of 41°C and that is normal for them. d) Normal core temperature varies between individuals within the range of 97.0°F to 99.5°F.

Normal core temperature varies between individuals within the range of 97.0°F to 99.5°F. Core temperature is normally maintained within a range of 36.0°C to 37.5°C (97.0°F to 99.5°F). -A core temperature greater than 41°C (105.8°F) or less than 34°C (93.2°F) usually indicates that the body's thermoregulatory ability is impaired. - Body heat is generated in the tissues of the body, transferred to the skin surface by the blood, and then released into the environment surrounding the body. -The thermoregulatory center regulates the temperature of the deep body tissues, or "core" of the body, rather than the surface temperature. -Internal core temperatures reach their highest point in late afternoon and evening and their lowest point in the early morning hours.

A two-day postoperative patient's temperature was 98.5°F at 3:00 pm. At 6:00 pm, the unlicensed assistant notifies the nurse that the patient's temperature is 102.0°F. Which of the following actions should the nurse take? a) Offer the client a cold drink b) Notify the physican c) Increase intravenous fluid rate d) Document the temperature

Notify the physician The nurse should contact the physician, as the increase in the patient's temperature is outside of the normal range and/or the normal diurnal variation in temperature.

Blood Count

Number of blood cells and their structural and functional characteristics.

A client is said to be in the chill stage of the fever process when the nurse does which of the following? a) Observes the client is sweating b) Observes piloerection on the skin c) Determines the client will benefit from a cool sponge bath d) Administers an antipyretic medication

Observes piloerection on the skin During the second stage or chill, there is the uncomfortable sensation of being chilled and the onset of generalized shaking (rigors). -Vasoconstriction and piloerection usually precede the onset of shivering. At this point, the skin is pale and covered with goose flesh. -Sweating is an indication of the third stage or flushing. -The other options are not descriptive of stages but rather interventions.

Phenylketonuria is abbreviated as ___.

PKU

Arthralgia

Pain in a joint

Which of the following patients would be diagnosed with wasting syndrome? a) Patient with AIDS with pneumonia, chronic fever, and chronic fatigue b) Patient with HIV a CD4+ T-cell count of 1000 cells/μL, Kaposi's sarcoma, and nausea and vomiting for more than 2 weeks c) Patient with chronic fatigue syndrome, no fever and herpes simplex, and candidiasis around the oral mucosa d) Patient with HIV, fever, diarrhea, and significant involuntary weight loss

Patient with HIV, fever, diarrhea, and significant involuntary weight loss A patient diagnosed with wasting syndrome (an AIDS-defining illness) would have chronic fever, diarrhea, and a significant involuntary weight loss (usually more than 10%) without an opportunistic infection. Pneumonia, Kaposi's sarcoma, and herpes simplex and candidiasis are all opportunistic infection.

Which of the following helps the innate immune system differentiate between body cells and foreign substances? a) Distinct antigenic properties allow the T cells to isolate specific organisms. b) Chemokines attract monocytes to sites of chronic inflammation. c) Pattern recognition receptors identify structures not shared by body tissues. d) Colony-stimulating factors are triggered by the presence of activated lymphocytes.

Pattern recognition receptors identify structures not shared by body tissues. Thus, the CD4+ TH1 cell controls and coordinates host defenses against certain intracellular pathogens, a function that helps to explain why a decreased CD4+ TH1 count in persons with acquired immunodeficiency syndrome (AIDS) places them at high risk for intracellular pathogen infections. If the teenager has a healthy immune system, high school should not place him or her at high risk for infection. A WBC of 8000 is normal. Radioactive iodine attacks the thyroid gland and does not usually interfere with immunity.

A client in the acute stage of inflammation will experience vasodilation of the arterioles and congestion in the capillary beds. The nurse would assess the client's skin for: a) Coolness b) Increased bacterial infection c) Redness d) Decreased sensation at the site

Redness Vasodilation of the arterioles and congestion of the capillary beds result in an increased pooling of blood leading to redness. The site would also have increased painful sensation and be warmer to touch. It would not result in an increase in bacterial load.

"A shift to the left"

Refers to WBC differential. Talks about the increase in immature neutrophils seen in severe infections.

How does Necrosis work?

cells swell and rupture, inflammation results

______ is a rare autosomal recessive disorder caused by an inborn error of metabolism that affects the way the body breaks down protein.

Phenylketonuria

Which one of the following is a common opportunistic infection in AIDS patients? a) Chancroid b) Myalgia c) Syphilis d) Pneumocystis jiroveci pneumonia (PCP)

Pneumocystis jiroveci pneumonia (PCP) PCP is a very common opportunistic infection in individuals with AIDS. It is a common bacterial found in households and does not routinely effect someone with a healthy immune system. Chancroid and syphilis are infections that do not occur commonly in the population in general including those with AIDS. Myalgia is not an infection, but it is muscle pain, a common symptom of an infection

Which of the following patients is most likely to have impairments to the wound healing process? A patient with: a) A diagnosis of multiple sclerosis and consequent impaired mobility. b) Congenital heart defects and anemia. c) Poorly controlled blood sugars with small blood vessel disease. d) Chronic obstructive pulmonary disease.

Poorly controlled blood sugars with small blood vessel disease. Diabetes mellitus is strongly associated with impaired wound healing. The other noted pathologies are less causative of deficiencies in the healing process.

Tensile strength of primary intention vs. secondary intention wound 3 months after injury

Primary intention: 70-80% of pre-injury strength Secondary: less

Primary vs. Secondary Intention in cutaneous wounds

Primary: less loss of tissue and contamination Ex: sutured surgical incision Secondary: greater loss of tissue and contamination, slow healing Ex: burns, large surface wounds

Opsonization

Process of making cells such as bacteria more susceptible to the action of phagocytes.

Which of the following is the priority intervention for a nurse caring for a patient with wasting syndrome from HIV? a) Administer antidiarrheal medications b) Monitor daily weight c) Promote nutritional intake d) Provide moisturizing skin care

Promote nutritional intake Wasting syndrome is defined as an AIDS-related illness with the manifestations of involuntary weight loss of at least 10% of baseline, diarrhea or chronic weakness and fever, but no other infections or neoplasms. It is treated with nutritional interventions such as oral supplements or enteral or parenteral nutrition.

Which can cause a fever response? - Hepatic encephalopathy - Thrombosis - Pulmonary Embolism

Pulmonary Embolism.

The nurse needs to assess a 1-year-old child for fever. Which approach will produce the most accurate reading? a) Rectal b) Axillary c) Oral d) Forehead

Rectal Measurement of core body temperature is important when evaluating fever. The rectal route is considered the most accurate. In adults and older children, the oral route is lower, but still accurate; however, in young children the oral route may be unreliable. -Forehead thermometers can predict trends, but are not as accurate as other routes. -The axillary route requires up to 10 minutes for the temperature to register appropriately.

The nurse is attempting to obtain the most accurate core body temperature of a patient. Which of the following methods should the nurse use? a) Oral b) Rectal c) Axillary d) Ear based

Rectal The rectal temperature is considered the most accurate parameter for measuring core body temperature compared to oral, axillary, and ear based.

While sponging a client who has a high temperature, the nurse observes the client begins to shiver. At this point, the priority nursing intervention would be to: a) Increase the room temperature by turning off the air conditioner and continue sponging the client with warmer water b) Administer an extra dose of aspirin c) Place a heated electric blanket on the client's bed d) Stop sponging the client and retake a set of vital signs

Stop sponging the client and retake a set of vital signs Modification of the environment ensures that the environmental temperature facilitates heat transfer away from the body. Sponge baths with cool water or an alcohol solution can be used to increase evaporative heat losses. -More profound cooling can be accomplished through the use of a cooling blanket or mattress, which facilitates the conduction of heat from the body into the coolant solution that circulates through the mattress. - Care must be taken so that cooling methods do not produce vasoconstriction and shivering that decrease heat loss and increase heat production.

A client with pneumonia is admitted with these vital signs: temperature 99.7ºF, pulse 80 beats/min, respirations 18/minute, and BP 120/80 mmHg. Which set of vital signs does the nurse anticipate when the client begins to shiver and requests another blanket several hours later?

T 100.9ºF, P 90/min, R 20/min, BP 126/80 mmHg During the chill phase of fever, the client feels cold and may experience pale skin with goosebumps, but the temperature is rising. When the body reaches the new set point, shivering will stop, and flushing will begin.

A client has immunity that was mediated by specific T lymphocytes and provides defense against intracellular microbes such as viruses. This immunity is recognized as: a) T lymphocytes b) Antibodies c) Suppressor B cells d) Histamines

T lymphocytes Cellular immunity is mediated by cells, such as specific T lymphocytes, and defense against intracellular microbes such as viruses. Histamine is released when the binding of antigen to mast cell- or basophil-bound IgE triggers a response. B cells mediate humoral immunity, and antibodies are produced by cells called B lymphocytes.

Which of the following explains how T lymphocytes and B lymphocytes differ? a) T lymphocytes are helpers that assist B lymphocytes to amplify the lysis of antigens. b) T lymphocytes secrete antibodies; B lymphocytes provide cellular response to antigens. c) T lymphocytes recognize bacteria; B lymphocytes recognize viruses and cancer. d) T lymphocytes mature in the thymus gland; B lymphocytes mature in the bone marrow.

T lymphocytes mature in the thymus gland; B lymphocytes mature in the bone marrow. Explanation: T lymphocytes mature in the thymus gland and provide cell-mediated immunity. T lymphocytes activate other lymphocytes and phagocytes. B lymphocytes mature in the bone marrow and are responsible for forming the antibodies that provide humoral immunity. Both are amplified by cytokines

Scenario: 2 boys suffer from hypoxia : one with a normal body temp and one who is very cold- Which one will have more cell swelling?

The one who is very cold due to vasoconstriction and an increased viscosity of blood

A pregnant client who is positive for HIV is concerned about her infant acquiring the virus. The best response by the nurse would be that children born to mothers with HIV infection are considered uninfected if: Select all that apply. a) They become HIV antibody negative after 18 months of age b) They are delivered via C-section c) They have no other laboratory evidence of HIV infection d) They become HIV antibody positive after 20 months of age e) They have not met the surveillance case definition criteria for AIDS in children

They become HIV antibody negative after 18 months of age • They have no other laboratory evidence of HIV infection • They have not met the surveillance case definition criteria for AIDS in children Children born to mothers with HIV infection are considered uninfected if they become HIV antibody negative after 18 months of age, have no other laboratory evidence of HIV infection, and have not met the case definition criteria for AIDS in children. Infants born to HIV-infected women can be HIV antibody positive by ELISA up until 18 months of age even though they are not infected with HIV. The route of delivery cannot guarantee chance of transmission.

A 33-year-old client is brought into the emergency room with a core temperature of 39°C (102.2°F). The client is red in the face, chest, and back due to significant cutaneous vasodilation. The client is likely in which stage of fever? a) Fourth b) Second c) First d) Third

Third The first stage of a fever is marked by headache and body aches, the -second stage is marked by the chills, and the -third stage is the flush state. The -fourth stage is defervescence.

Dilation of superficial blood vessels occurs in fever why?

To increase the skin temp to dissipate the heat.

Which of the following serves as the rationale for AIDS treatment with antiretroviral therapy? a) Monotherapy with protease inhibitors b) Triple therapy with reverse transcriptase, protease, and fusion inhibitors c) Monotherapy with reverse transcriptase inhibitors d) Monotherapy with fusion inhibitors

Triple therapy with reverse transcriptase, protease, and fusion inhibitors Because different drugs act on various stages of the replication cycle, optimal treatment includes a combination of drugs, including reverse transcriptase inhibitors, protease inhibitors, fusion/entry inhibitors, and integrase inhibitors. Each type of agent attempts to interrupt viral replication at a different point.

Radiation

UV rays-disrupt intracellular bonds and cause sunburn which can lead to skin cancer

Erythrocyte Sedimentation Rate

Used to detect inflammation.

A client who has developed a fever is now complaining of a headache. The nurse would recognize this manifestation as a result from the:

Vasodilatation of cerebral vessels Headache is a common accompaniment of fever and is thought to result from the vasodilatation of cerebral vessels occurring with fever. The coronary arteries would not contribute/cause the headache.

A client who has developed a fever is now complaining of headache? Nurse would recognize this manifestation as a result from the...

Vasodilation of cerebral vessels.

Neutropenia

Viral infectiosn tend to produce a decrease in neutrophils and an increase in lymphocytes (lymphocytosis). This is called...

Example of Hyperplasia

Warts and prostate

Scenario: Mr.X had a stroke and blood flow is cut off to part of the brain- Why give him a calcium channel blocker?

We want to block calcium to prevent damage of cells and ATP depletion

innate immunity

cells/molecules that area already setup before infection and can work immediately as an barrier (the same every time, nonspecific)

A client who was exposed to hepatitis A at a local restaurant has recovered from the disease. At her annual physical, the client asks the health care provider if she should go to her health department and get the hepatitis A "shot." The best response, based on the concepts of adaptive immunity, by the health care provider would be: a) "I wouldn't since the vaccine can damage your liver." b) "No, since having an active case, you have already developed antigens against hepatitis A." c) "Of course. The virus changes every year." d) "Yes, because you could get a worse case the next time you are exposed."

b) "No, since having an active case, you have already developed antigens against hepatitis A." The adaptive immune system consists of two groups of lymphocytes and their products, including antibodies. Whereas the cells of the innate immune system recognize structures shared by classes of microorganisms, the cells of the adaptive immune system are capable of recognizing numerous microbial and noninfectious substances and developing a unique specific immune response for each substance. Substances that elicit adaptive immune responses are called antigens. A memory of the substance is also developed so that a repeat exposure to the same microbe or agent produces a quicker and more vigorous response. The hepatitis A virus does not change from year to year. The vaccine does not damage the liver; however, the active hepatitis A disease can

A sixth grade science teacher asks the students to explain the role of cilia in the lower respiratory tract. Which student response is the best? a) "Cilia help to warm the airways so that moisture in the air can neutralize any germs that get in our lungs." b) "These little hairs move germs trapped in mucous toward the throat so the body can cough them out." c) "Cilia help facilitate a chemical defense against germs by secreting an enzyme that will cement the germ to the lining of the airways." d) "Cilia can trap the microbes in one location so the body can grow scar tissue around them and wall them off so they can't cause disease."

b) "These little hairs move germs trapped in mucous toward the throat so the body can cough them out." Also in the lower respiratory tract, hair-like structures called cilia protrude through the epithelial cells. The synchronous action of the cilia moves many microbes trapped in the mucous toward the throat. The physiologic responses of coughing and sneezing further aid in their removal from the body. Chemical defenses against trapped microbes include the following: lysozyme, which is a hydrolytic enzyme capable of cleaving the walls of bacterial cells; complement, which binds and aggregates bacteria to increase their susceptibility to phagocytosis or disrupt their lipid membrane; and members of the collectin family of surfactant proteins in the respiratory tract. The best-defined function of the surfactants is their ability to opsonize pathogens, including bacteria and viruses, and to facilitate phagocytosis by innate immune cells such as macrophages.

Place the following in the sequence of the progression of HIV/AIDS on a cellular level. 1)The HIV virus is uncoded. 2)HIV attaches to the receptors on the CD4+ cell. 3)DNA synthesis occurs by reverse transcription. 4)Transcription of the DNA to form a single-strand messenger RNA. a) 3,4,1,2, b) 2,1,3,4 c) 1,2,3,4 d) 4,3,1,2

b) 2,1,3,4 The first step involves the binding of the virus to the CD4+ T cell. The second step allows for the internalization of the virus. After attachment, the virus envelope peptides fuse to the CD4+ t-cell membrane. Fusion results in uncoding of the virus. The third step consists of DNA synthesis by reverse transcription. The fourth step is integration. The fifth step involves transcription of the double-stranded viral CNA to form a singe-stranded messenger RNA

A child has a congenital condition in which the thymus gland is absent. What should the nurse include in the education of care of this patient to the parents? a) Observe and report any signs of bleeding. b) Observe and report signs and symptoms of infection. c) Do not immunize the child with inactive or live viruses. d) Report if the child has 2-3 loose stools in a day.

b) Observe and report signs and symptoms of infection. The thymus is essential to the development of the immune system because it is responsible for the production of mature, immunocompetent T lymphocytes.

Which one of the following is the main effect of HIV infection? a) Poor natural killer cell function b) Poor helper T-cell function c) Poor B-cell function d) Poor suppressor T-cell function

b) Poor helper T-cell function HIV infects a limited number of cell types in the body, including a subset of lymphocytes called CD4+T lymphocytes (also known as T-helper cells or CD4+T cells). The CD4+T cells are necessary for normal immune function. Among other functions, the CD4+T cell recognizes foreign antigens and helps activate antibody-producing B lymphocytes. The phagocytic function of monocytes and macrophages is also influenced by CD4+T cells.

The nurse understands that a positive ELISA test reported in a baby born to a woman who is HIV positive indicates which of the following? a) The baby has AIDS. b) The baby has antibodies from the mother and it is uncertain whether the baby is infected with HIV. c) The baby has HIV infection. d) The baby is not infected with HIV.

b) The baby has antibodies from the mother and it is uncertain whether the baby is infected with HIV. The diagnosis of HIV infection in children born to HIV-infected mothers is complicated by the fact that infants have the maternal anti-HIV IgG antibody for approximately 6 months. Consequently, infants born to HIV-infected women can be HIV antibody positive by ELISA for up to 18 months of age even though they are not infected with HIV

A teenage client is exposed to a person infected with chicken pox. After two weeks, the client has not contracted the virus. How is this possible? Select all that apply. a) Only children are at risk of contracting chicken pox; teens and adults are immune. b) The client was vaccinated for chicken pox. c) The client was previously exposed to chicken pox. d) The client is naturally immune from birth. e) The chicken pox virus is not contagious.

b) The client was vaccinated for chicken pox. c) The client was previously exposed to chicken pox. Active immunity is acquired when the host mounts an immune response to an antigen either through the process of vaccination or from environmental exposure. It is called active immunity because it requires the host's own immune system to develop an immunological response including the development of memory. Passive immunity transferred from mother to fetus only lasts a few weeks to months following birth.

One way to diagnose Chronic Hex-A deficiency is to look for a characteristic ______ ______ spot on the ______.

cherry red, retina

A mother is diagnosed with a bacterial infection and is worried that her newborn infant will also contract the infection. Which of the following statements should the nurse include in the teaching plan for the client? a) Since the newborn immune system is not mature your infant cannot contract the infection. b) Your newborn has maternal IgG antibodies that were transferred through the placenta before birth, providing some protection from infection. c) Since you are not immune to this infection, your infant is also at risk for contracting the infection. d) Your infant will require a vaccination to avoid getting the infection.

b) Your newborn has maternal IgG antibodies that were transferred through the placenta before birth, providing some protection from infection. Passive immunity is immunity transferred from another source. The most common form of passive immunity is that conferred from mother to fetus. During fetal development, maternal IgG antibodies are transferred to the fetus via the placenta. After birth, the neonate also receives IgG antibodies from the mother in breast milk or colostrum. Therefore, infants are provided with some degree of protection from infection for approximately 3 to 6 months, giving their own immune systems time to mature.

microbe

bacteria causing disease

pathogens

bacteria or virus that causes disease

"Congenital defects" are also known as...

birth defects

Onset of NF-1 may be evident at ______ and nearly always by age ___.

birth, 10

One way to diagnose Chronic Hex-A deficiency is to perform ______ ______ to check levels of ______.

blood tests, Hex-A

One diagnostic technique for Marfan Syndrome is a ______ test which detects ______ of ______.

blood, lack, fibrillin

Heat

burn and heatstroke; increase in cell metabolism, inactivate temperature sensitive enzymes, disrupt cell membrane

Which of the following would be the best treatment option to prevent perinatal transmission of HIV antibodies to a fetus from the HIV-positive mother? a) Administration of efavirenz starting within the first trimester of pregnancy b) Administration of a fusion inhibitor immediately on diagnosis of pregnancy c) Administration of zidovudine to the mother during pregnancy, labor, and delivery d) Administration of zidovudine to the newborn immediately after delivery

c) Administration of zidovudine to the mother during pregnancy, labor, and delivery The administration of Zidovudine has proven to be effective in lowering the perinatal transmission by two-thirds as compared to those who do not take the medications. Administration of Zidovudine to the infant would not be therapeutic. Efavirenz, a fusion inhibitor, is contraindicated related to its teratogenetic effects in the first trimester.

The nurse knows high incidences of infectious illnesses among the older adults who reside in a long term care facility are most likely to have diminished immune capacity because of: a) Altered function in peripheral lymphocytes b) Over-expression of cytokines and receptors c) Decreased numbers and responsiveness of T lymphocytes d) Decreased antigen recognition by B lymphocytes

c) Decreased numbers and responsiveness of T lymphocytes Although this phenomenon is not well understood, increasing proportions of lymphocytes become unresponsive with age, and CD4+ T lymphocytes are the most severely affected. B lymphocytes recognize more antigens, not fewer, and expression of cytokines and their cellular receptors decreases

A 10 year old child with strep throat asks the nurse, "why are there large bumps [lymph nodes] on their neck when their throat gets sore?" The nurse replies that lymph nodes: a) Bring all kind of good cells to your throat so that they can wall the strept off and keep the germs from getting any food or water. b) Help your tonsils get bigger with cells that will bring immune cells into your throat to prevent any other infections. c) Help your body fight off infections by allowing special cells [lymphocytes and macrophages] move through the lymph chain and engulf and destroy germs. d) Brings in cells into the lymph node [your bump] to stop the germs from going anywhere else in the body.

c) Help your body fight off infections by allowing special cells [lymphocytes and macrophages] move through the lymph chain and engulf and destroy germs.

A patient is being treated for an anaphylactic reaction after eating shrimp. Which of the following classes of antibodies mediates the anaphylaxis? a) IgA b) IgM c) IgE d) IgG

c) IgE Anaphylaxis is a catastrophic, systemic life-threatening IgE-mediated hypersensitivity reaction associated with the widespread release of histamine into the systemic circulation that produces massive vasodilation, hypotension, arterial hypoxia, and airway edema.

A client experiences an allergic reaction. Select the immunoglobulin that would bind to mast cells and release histamine. a) IgG b) IgA c) IgE d) IgD

c) IgE IgE responds to allergic exposures and parasitic infections. IgA is the primary defense against local infections in mucosal tissues. IgG protects against bacteria, toxins, and viruses and activates the complement system. IgD acts as an antigen receptor for initiating the B cells

The nurse is caring for a client who has AIDS and suffers from wasting syndrome. Which of the following is the priority nursing consideration for this client? a) Pain b) Oxygenation c) Nutrition d) Skin care

c) Nutrition The wasting syndrome is an AIDS-defining illness and is common in people with HIV infection or AIDS. Wasting syndrome is characterized by involuntary weight loss of at least 10 percent of baseline. Treatment for wasting includes nutritional interventions

While explaining immunity to a client, the nurse responds, "The body's internal organs are protected from pathogens because: a) We have special glands that can secrete cytokines on a moment's notice." b) The tonsils store a large amount of natural killer cells at that location." c) Our mucosal tissue contains all the necessary cell components to fight a pathogen with an immune response." d) The actions of the cytokines in the mouth can act on different cell types at the same time it is fighting pathogens."

c) Our mucosal tissue contains all the necessary cell components to fight a pathogen with an immune response." Secondary lymphoid tissues contain all the necessary cell components (i.e., T cells, B cells, macrophages, and dendritic cells) for an immune response. Because of the continuous stimulation of the lymphocytes in these tissues by microorganisms constantly entering the body, large numbers of plasma cells are evident. Immunity at the mucosal layers helps to exclude many pathogens and thus protects the vulnerable internal organs. Although cells of both the innate and adaptive immune systems communicate critical information by cell-to-cell contact, many interactions and effector responses depend on the secretion of short-acting soluble molecules called cytokines. The actions of cytokines are often pleiotropic and redundant. Pleiotrophism refers to the ability of a cytokine to act on different cell types.

HIV is considered to be a retrovirus because: a) it reproduces at a rapid rate. b) it converts to a primitive form of virus when duplicated. c) it carries its genetic information in ribonucleic acid (RNA) rather than deoxyribonucleic acid (DNA). d) it carries a genetic marker for a previously discovered virus that was a source of an epidemic in an earlier time period.

c) it carries its genetic information in ribonucleic acid (RNA) rather than deoxyribonucleic acid (DNA). Like other retroviruses, HIV carries its genetic information in ribonucleic acid (RNA) rather than deoxyribonucleic acid (DNA). In the process of taking over the CD4+ T-cell, the virus attaches to receptors on the CD4+ cell, fuses to and enters the cell, incorporates its RNA into the cell's DNA, and then uses the CD4+ cell's DNA to reproduce large amounts of HIV, which are released into the blood.

The nurse understands that the best way for a health care worker to protect against the transmission of HIV is to: a) use proper hand-washing techniques before and after contact with all clients. b) use contact-isolation on all clients known to be HIV positive. c) use universal precautions on all clients. d) use blood and body fluid precautions on clients known to be HIV positive.

c) use universal precautions on all clients. Universal Blood and Body Fluid Precautions should be used in encounters with all people in the health care setting since it should be assumed that any person may have a transmissible infection.

Example of Dysplasia

cancer

Tumors resulting from NF-1 may become ______, requiring ______ ______ and ______.

cancerous, radiation therapy, chemotherapy

A client who is scheduled for orthopedic surgery has expressed concern about the risk of malignant hyperthermia. The nurse should reassure the client that the anesthesiologist will: a) administer antipyretics intravenously at the first sign of the problem. b) administer dantrolene 30 to 60 minutes before surgery to prevent the problem. c) intubate the client in order to prevent malignant hyperthermia d) carefully assess the client's genetic risk of the problem.

carefully assess the client's genetic risk of the problem. Family history is a significant factor when appraising a client's risk of malignant hyperthermia. Dantrolene is only administered if active treatment is needed. Antipyretics are ineffective. Intubation is necessary during surgery but does not prevent malignant hyperthermia.

Manifestations of NF-2 include early onset of ______.

cataracts

Injurious agents

causes stress damage

Neurofibromatosis leads to uncontrolled ______ ______ causing ______ ______

cell division, tumor formation

The mating of two related individuals is called "______ ______," or ______.

consanguineous mating, inbreeding

One symptom of Tay-Sachs Disease is the affected individual may experience ______.

convulsions

A patient will be receiving a bone graft from an unrelated individual. Which of the following types of graft does the nurse inform the patient that he or she will be receiving? a) Autograft b) Syngeneic graft c) Monogeneic graft d) Allograft

d) Allograft Transplanted tissue can be categorized as an autologous graft or autograft if donor and recipient are the same person, syngeneic graft if the donor and recipient are identical twins, and allogeneic or allograft if the donor and recipient are unrelated but share similar human leukocyte antigen tissue expression.

The daughter of a 79-year-old woman asks the nurse why her mother gets so many infections. The daughter states, "My mother has always been healthy, but now she has pneumonia. Last month she got cellulitis from a bug bite she scratched. The month before that was some other infection. How come she seems to get sick so often now?" What is the nurse's best response? a) Your mother just seems to be prone to getting infections. b) Your mother gets infections frequently because she wants attention from you. c) About the time we are 75 or 76 years old, our immune system quits working. d) As people get older, their immune system does not respond as well as it did when they were younger.

d) As people get older, their immune system does not respond as well as it did when they were younger. Aging is characterized by a declining ability to adapt to environmental stresses. One of the factors thought to contribute to this problem is a decline in immune responsiveness. This includes changes in cell-mediated and humoral immune responses. Elderly persons tend to be more susceptible to infections, have more evidence of autoimmune and immune complex disorders than younger persons, and have a higher incidence of cancer. None of the other answers are true or acceptable.

Which of the types of T cells is responsible for destroying pathogens by punching holes in their cell membrane and by secreting cytokines/lymphokines? a) Helper T cells b) Regulatory T cells c) Memory T cells d) Cytotoxic T cells

d) Cytotoxic T cells Cytotoxic T cells destroy the pathogens. Regulatory T cells suppress immune response. Memory T cells are derived from normal T cells. The principal cells of the adaptive immune system are the T and B lymphocytes and antigen-presenting cells. T lymphocytes differentiate into helper T and regulatory T cells and cytotoxic T cells and provide cell-mediated immunity. CD4+ helper T cells serve as a trigger for the immune response and are essential for the differentiation of B cells into antibody-producing plasma cells and the differentiation of T lymphocytes into CD8+ cytotoxic T cells.

Natural killer cells are specialized lymphocytes that are one of the major parts of which immunity? a) Humoral b) Adaptive c) Cell mediated d) Innate

d) Innate The major components of innate immunity are the skin and mucous membranes; phagocytic cells (mainly neutrophils and macrophages); specialized lymphocytes called NK cells; and several plasma proteins, including the proteins of the complement system. Adaptive, humoral, and cell-mediated immunities do not use NK cells.

While caring for a pediatric client admitted with a viral infection, the nurse knows that which type of cell will be the child's primary defense against the virus? a) Bradykinin b) Leukotrienes c) Complement d) Natural killer (NK) cells

d) Natural killer (NK) cells

The treatment prescribed for an autoimmune disorder is primarily dependent upon what? a) The age and gender of the client b) The presence of existing chronic disorders and the client's medical history c) Corticosteroids is always the first line of treatment. d) The current manifestations of the disease and the mechanisms that cause the disease process

d) The current manifestations of the disease and the mechanisms that cause the disease process Treatment of autoimmune disorders is dependent upon the magnitude of the presenting manifestations and underlying mechanisms of the disease process. Since in many cases the pathophysiologic mechanisms are not always known, treatment may be purely symptomatic. While corticosteroids are often use, they are not always the first line of treatment. The client's age and medical history can be factors in treatment choices but are not the primary consideration.

Increased intracellular calcium causes

damge to cell organelles

Reasons for Necrosis : Caseous Necrosis

dead cells persist indefinitely such as in tuberculosis

Atrophy

decrease in size

In an autosomal recessive disorder, if one ______ gene and one ______ gene is passed to the child, that child is said to be a "______".

defective, normal, carrier

natural killer cells

defends against various viruses and cancer cells (tumor cells & infected cells)

The most critical factor in preventing mental retardation from PKU is beginning treatment ______ and continuing during the ______ years of life.

immediately, earliest

Autosomal recessive disorders include almost all ______ ______ in ______.

inborn errors, metabolism

In Adult-Onset Tay-Sachs Disease, affected individuals live ______ ______ lives, although most spend their adulthood in a ______.

full adult, wheelchair

Hex-A is a protein that helps break down ______ in ______ tissue.

gangliosides, nerve

Congenital defects may be caused by what two types of factors?

genetic and environmental

Mutations that are transmitted to the offspring involve what cell type?

germ cells

Manifestations of NF-1 include two or more ______ on the ______ of the ______.

growths, iris, eye

Manifestations of NF-1 include an abnormally large ______ ______.

head circumference

Tumors resulting from NF-2 may cause ______, ______, and ______ loss.

headaches, tinnitus, hearing

Manifestations of Marfan Syndrome may include floppy ______ ______.

heart valves

Death in result of Marfan Syndrome is most often caused by ______ failure or ______ ______ aneurysm.

heart, ruptured aortic

One way to diagnose PKU is a routine ______ ______ at birth.

heel stick

An example of a stressed cell filling up with calcium salts is

hyperparathyroid (hypercalcemia)

Reasons for Necrosis : Coagulation

hypoxic areas (Lack of oxygen)

Hypertrophy

increase in size

Which clients are showing manifestations of infection? Select all that apply. a) A 50-year-old, temperature 36.2°C (97.3°F), heart rate 65 beats/min, muscle aches b) A 25-year-old, temperature 40°C (104°F), sweating, shivering, states generalized pain c) A 5-year-old, temperature 36.8°C (98.2°F), talkative, actively engaged in playing d) A 2-month-old, temperature 38.3°C (100.4°F), lethargy, poor feeding, and cyanosis e) A 75-year-old, temperature 37.3°C (99.2°F), declining mental status, weakness and fatigue

• A 25-year-old, temperature 40°C (104°F), • A 2-month-old, temperature 38.3°C (100.4°F), lethargy, poor feeding, and cyanosis • A 75-year-old, temperature 37.3°C (99.2°F), declining mental status, weakness and fatigue An older adult with an infection may have a minimal rise in temperature, but exhibit changes in mental status, weakness, fatigue, and weight loss. An infant younger than three months may have a relatively mild fever, but a serious infection. An adult with a high fever will exhibit sweating and chills. Aches and pains may occur with shivering and the infectious illness. - A client with a temperature within the normal range, exhibiting no other signs of change, is not considered to have an infection.

A nurse assessing an older adult for signs and symptoms of infection in the absence of a fever should assess for which of the following? Select all that apply. a) Decreased mental status b) Change in fuctional capacity c) Fatigue d) Depression e) Weight gain

• Decreased mental status • Change in fuctional capacity • Fatigue Signs and symptoms of infection in an older adult in the absence of a fever include decreases in mental status and functional capacity, fatigue, weight loss, and weakness.

Which of the following lab results confirm the client has developed an acute-phase inflammatory response? Select all that apply. a) C-reactive protein (CRP) 10.0 mg/L (high). b) Leukocytes (WBC) 18.7 cells/μL (high). c) Fibrinogen level 1.5 g/L (normal). d) Erythrocyte sedimentation rate (ESR) 175 mm/h (high). e) Red blood cell count (RBC) 3.11 cells/μL (low).

• Erythrocyte sedimentation rate (ESR) 175 mm/h (high). • Leukocytes (WBC) 18.7 cells/μL (high). • C-reactive protein (CRP) 10.0 mg/L (high). During the acute-phase response, the liver dramatically increases the synthesis of acute-phase proteins such as fibrinogen, C-reactive protein (CRP), and serum amyloid A protein (SAA) that serve several different defense functions. -The synthesis of these proteins is stimulated by cytokines, especially TNF-α, IL-1 (for SAA), and IL-6 (for fibrinogen and CRP). -The accelerated erythrocyte sedimentation rate (ESR) that occurs in disease conditions is characterized by the systemic inflammatory response. -Leukocytosis, or the increase in white blood cells, is a frequent sign of an inflammatory response, especially those caused by bacterial infection. - In acute inflammatory conditions, the white blood cell count commonly increases from a normal value of 4000 to 10,000 cells/μL.

A newly diagnosed HIV positive adolescent has blood work drawn which includes a CD8 T cell count. The nurse knows which of the following functions of CD8 T cells listed below will assist the adolescent's immune system in fighting off the viral attack? Select all that apply. a) Booster antigen-antibody response. b) Removal of foreign material from lymph before it enters the blood. c) Cause allergens to surround the virus. d) Release of destructive enzymes. e) Trigger intracellular programmed death.

• Release of destructive enzymes. • Trigger intracellular programmed death. Explanation: The primary function of Cytotoxic T (CD8) cells is to monitor the activity of all cells in the body and destroy any that threaten the integrity of the body. The CD8 cells destroy target cells by releasing cytolytic enzymes, toxic cytokines, and pork-forming molecules or by triggering membrane molecules and intercellular apoptosis. Removal of foreign material from lymph before it enters the blood is the function of lymph nodes.

A client presents with a temperature of 38.8°C (101.8°F), a racing heart, fatigue, and an upset stomach after working outside in a hot day. Which assessment findings suggest fever rather than hyperthermia as a cause of the elevation in the man's temperature? Select all that apply. a) Pallor b) Cognitive changes c) Shivering d) Dizziness e) Moist skin

• Shivering • Pallor Shivering is a response to a signal for increased heat production. It would only occur on a hot day if the set point for temperature regulation were increased, as in the case of fever but not hyperthermia. Similarly, pallor is more closely associated with an infection causing fever than hyperthermia. -Moist skin, cognitive changes, and dizziness can occur with either fever or hyperthermia.

Which of the following characteristics correspond with innate immunity? Select all that apply. a) Immediate response to antigen invasion b) Large specific recognition system for microbes and antigens c) Cellular components include T and B lymphocytes d) Similar immune response for each exposure e) Antibodies are part of the molecular response

• Similar immune response for each exposure • Immediate response to antigen invasion Explanation: Innate immunity is immediate, but it is limited to certain general classes of microbes instead of to specific organisms. The response to each infection is similar; some defenses are the epithelium of the skin and mucous membranes along with inflammation and fever. Cellular components of innate immunity include phagocytes, natural killer cells, and dendritic cells. Cytokines, complement proteins, acute-phase proteins, and soluble mediators are molecular components of the innate immunity.

A client has presented to the emergency department after he twisted his ankle while playing soccer. Which assessment findings are cardinal signs that the client is experiencing inflammation? Select all that apply. a) The ankle is bleeding b) The ankle is warmer than the unaffected ankle c) The client is experiencing pain d) The ankle appears to be swollen e) The client's ankle is visibly red

• The ankle is warmer than the unaffected ankle • The client is experiencing pain • The ankle appears to be swollen • The client's ankle is visibly red The cardinal signs of inflammation are rubor (redness), tumour (swelling), calor (heat), and dolor (pain). -Bleeding is not among the cardinal signs.


Kaugnay na mga set ng pag-aaral

APES Unit 2 AP Classroom Questions

View Set

Chapter 15: Shock and Resuscitation

View Set

Chp 7: Metabolism: From Food to Life

View Set

Principles of Real Estate Chapter 9

View Set

Reading plus: The Science of Slime {3 ⭐️'s}

View Set

Chapter 10 Operations: Quality Control

View Set